3
$\begingroup$

I am very interested in the maximum number of triangles could a connected graph with $n$ vertices and $m$ edges have. For example, if $m\leq n−1$, this number is $0$, if $m=n$, this number is $1$, if $m=n+1$, this number is $2$, and if $m=n+2$, this number is $4$.

$\endgroup$
2
  • 1
    $\begingroup$ If we omit the restriction of connectedness, see this question: math.stackexchange.com/questions/823481/… Also note that we can achieve connectedness with a few number of edges, so the order of magnitude of the number of triangles is the same without your restriction. $\endgroup$ Jun 22, 2015 at 10:55
  • $\begingroup$ @Soltész I want to get a good bound in terms of $m-n+1$. I believe this number could be bounded by $(m-n+1)^2$ roughly. $\endgroup$ Jun 22, 2015 at 11:10

2 Answers 2

8
$\begingroup$

It is a bound and since it is very long, I wrote it an answer, may be it can be helpful.

Let $G$ be a connected graph with $n$ vertices and $m$ edges. Suppose the eigenvalues of this graph are $\lambda_1\geq \lambda_2\geq\ldots\geq\lambda_n$. We know that $\sum{\lambda_i^3}=6\Delta_G$, where $\Delta_G$ counts the total number of triangles of the graph $G$.

Also,we have:

$$\lambda_1\leq\sqrt{2m-\delta(n-1)+\Delta(\delta-1)}.$$

Since your graph is connected, we can set $\delta=1$ and obtain: $$\lambda_1\leq\sqrt{2m-n+1}.$$

So we have:

$$\Delta_G\leq\frac{n}{6}(2m-n+1)^{\frac{3}{2}},$$

as you wanted in your comments.

Actually, you can get more information from this method since we exactly know when the upper inequalities which I used are equality for which graphs. You can search for "SHARP UPPER BOUNDS OF SPECTRAL RADIUS OF GRAPHS" or similar keywords.

$\endgroup$
2
  • $\begingroup$ Thanks, this method is very helpful. This bound can be easily improved as $\Delta_G\leq\frac{n-1}{6}(2m-n+1)^{\frac{3}{2}}$. $\endgroup$ Jun 27, 2015 at 8:07
  • $\begingroup$ This can be further improved with $\Delta_G \leq m^{3/2}$. $\endgroup$
    – orezvani
    Feb 16, 2017 at 2:58
3
$\begingroup$

This question (together with massive generalizations) is answered in I. Rivin's 2001 paper.

$\endgroup$
1
  • $\begingroup$ The paper seems interesting. It has some trivial results, but one up vote for the very nice style of writing. $\endgroup$
    – Shahrooz
    Apr 16, 2016 at 9:28

Your Answer

By clicking “Post Your Answer”, you agree to our terms of service and acknowledge you have read our privacy policy.

Not the answer you're looking for? Browse other questions tagged or ask your own question.